Wikipedia:Reference desk/Archives/Science/2013 February 8

From Wikipedia, the free encyclopedia
Science desk
< February 7 << Jan | February | Mar >> February 9 >
Welcome to the Wikipedia Science Reference Desk Archives
The page you are currently viewing is an archive page. While you can leave answers for any questions shown below, please ask new questions on one of the current reference desk pages.


February 8[edit]

Camera quality as a function of weight[edit]

When taking a photo using a DSLR camera , is there a difference between a heavy camera and a light camera if the photo is taken handheld and all other factors being the same? — Preceding unsigned comment added by 149.135.147.66 (talk) 02:02, 8 February 2013 (UTC)[reply]

I'm no expert, but in film SLRs, heavier is generally better because of all of the film mechanism needing to get it in the correct place, etc. In a DSLR a heavier camera might be better to damp out some of the jiggling when it is hand held. Bubba73 You talkin' to me? 04:01, 8 February 2013 (UTC)[reply]
Maybe in a certain range, but there's also a point where things become so heavy we lose the ability to hold them steady. StuRat (talk) 04:23, 8 February 2013 (UTC)[reply]
This is where tripods or monopods come in. ←Baseball Bugs What's up, Doc? carrots→ 04:32, 8 February 2013 (UTC)[reply]
But the OP specifically said handheld which I would take to mean without using a tripod or monopod or other supporting device. They also said 'all other factors being the same' which is a big unclear but I take to mean that they're excluding the heavier camera possibly having a better lens, CCD and whatever else. Nil Einne (talk) 06:02, 8 February 2013 (UTC)[reply]
Plenty of sports photographers use monopods on their hand-helds, to help them steady the camera, especially on zoom-in shots. ←Baseball Bugs What's up, Doc? carrots→ 14:00, 8 February 2013 (UTC)[reply]
But how much does it help when the problem is the camera is too heavy? Nil Einne (talk) 14:42, 8 February 2013 (UTC)[reply]
The monopod (and hence the surface it's resting on) absorbs much of the weight of the camera, hence it's easier to keep steady. ←Baseball Bugs What's up, Doc? carrots→ 22:20, 8 February 2013 (UTC)[reply]
The pictures that I take my fixed lens compact film camera, are much crisper than those I take with the compact digital. With the film camera, I look through the view finder and the camera sits against my face, which helps to hold it still. With the digital, I have to hold it at arms length to see the viewing screen and it is very difficult to hold the camera still. The pictures that I take with the film SLRs are the crispest of all, as the cameras have size and you can get a proper grip. With small cameras, I have to hold them in my finger tips, otherwise part of the lens, or sensor, or view finder is obscured. I don't have a DSLR but I imagine the same it true. --TrogWoolley (talk) 09:30, 8 February 2013 (UTC)[reply]
The mechanics of holding cameras is probably part of the resolution difference, but compact digital cameras have poor resolution. Bubba73 You talkin' to me? 02:48, 10 February 2013 (UTC)[reply]

Anti-cancer molecule TRAIL[edit]

TRAIL was recently in the news concerning a new approach to fight cancer. I have a few questions about it:

  • Which cells produce TRAIL, and under what circumstances?
  • Does TRAIL act on the receptors of the cell that produced it, or on some other cell?
  • Is TRAIL a transmembrane protein, or is it released into the extracellular fluid? (Our article states that it has "characteristics of a transmembrane protein".)

If the answer to any of these questions is "not currently known", that would be helpful too. Thanks, AxelBoldt (talk) 04:11, 8 February 2013 (UTC)[reply]

The NCBI is a reliable source and has a page about TRAIL. TRAIL is expressed in most tissues, in various forms, but appears to cause apoptosis of transformed and tumor cells more than others. As a cytokine, it is secreted. Like TNF, it causes inflammation. -- Scray (talk) 04:23, 8 February 2013 (UTC)[reply]

MTBE[edit]

Now that MTBE has been mostly phased out as a gasoline additive, are there any large-scale uses for it left? 24.23.196.85 (talk) 06:38, 8 February 2013 (UTC)[reply]

As an additive to petrol or gasoline? That's what the article says.... So does this press release [1] Nil Einne (talk) 06:52, 8 February 2013 (UTC)[reply]
I thought that "petrol" is in fact the British word for gasoline? 24.23.196.85 (talk) 07:19, 8 February 2013 (UTC)[reply]
It is. That's why I said 'or' rather then 'and'. MTBE is evidently still used to some extent in a few countries like India [2], Singapore [3] and Malaysia [4] where the word petrol would be preferred. It's also used in some countries where the word gasoline might be preferred (well I'm not sure of this). I originally just used the word petrol, but decided to add gasoline to avoid confusion and argument. Nil Einne (talk) 07:41, 8 February 2013 (UTC)[reply]
MTBE is still used in Mali. There is a phase-out plan[5] but the old refineries are still running. Let me pull up my cheatsheet just in case anyone here is thinking of taking a vacation in Timbuktu...
Fuel of any type: Carburant.
Gasoline/petrol: L'essence or Essence
A sentence with Sélectionnez and carburant means "choose your fuel" Usually there is an octane number but you may see ordinaire or super.
A sentence with Validez and carburant means "confirm" (press "Val" to confirm.)
Unleaded: Essence sans plomb
Leaded: Essence au plomb
Diesel: Gazole or Gasoil
Liquid petroleum gas (LPG): GPL or Gépel
Heating oil: Fioul or sometimes Fuel
Crude Oil (and, I think, paraffin): pétrole
The last two come into play when you ask for petrol or fuel; they will say that they don't have any. Ask for gasoline and you may get essence or you may get gasoil.
Note: these are probably bad French.
BTW, there is an interesting story about how gasoline came to be called that: http://blog.oxforddictionaries.com/2012/04/the-origin-of-gasoline/ --Guy Macon (talk) 10:09, 8 February 2013 (UTC)[reply]
Are we talking about the same thing? The link you supplied is referring to Tetraethyllead which MTBE partially replaced. If they still haven't phased out Tetraethyllead in Mali, I'm not sure whether there are any concrete plans for phasing out MTBE (although the link you supplies is rather old). In any case, MTBE is still widely used in China. As I mentioned it is apparently used in India, Malaysia and even I think Singapore (I'm not 100% sure since the results get confused by level of discussion of production and blending in Singapore but Singapore is apparently a major importer). These are places (including China) which phased out tetraethyllead at least 13 years ago. It's also used in quite a few other Asian (including Arab) countries. I'm not sure if there is a clear phase out plan in any of these but as per the sources its use is apparently decreasing in some cases although because of price pressures not environmental or health concerns (apparently in China this includes a consumption tax [6]). However the sources suggest while production may not be increasing in worldwide terms (or at least not that much), it's not decreasing either. See also [7]. Either way all the evidence suggests MTBE is still widely used as a gasoline/petrol/whatever additive. P.S. Our article suggest TEL was phased out in Africa in 2006 although Algeria remains a holdout and there are a bunch of Asian countries and one arguably European country (Georgia) who are still using TEL. I suspect there are more who have some leaded petrol but don't really know. Nil Einne (talk) 12:36, 8 February 2013 (UTC)[reply]
You are right. I confused the two. (Note to self: next time, smoke crack after editing Wikipedia...) --Guy Macon (talk) 19:32, 8 February 2013 (UTC)[reply]
Wait a minute, how can Georgia be an "arguably European country" when in fact it's part of the USA? 24.23.196.85 (talk) 22:57, 8 February 2013 (UTC)[reply]
Nil Einne was referring to საქართველო (that's Sakartvelo for you illiterates). -- Jack of Oz [Talk] 01:04, 10 February 2013 (UTC)[reply]
Oh, that Georgia! In that case, they might well have different fuel specs from the ones in our country. So, getting back to the topic of MTBE, I gather that its only major use is as a fuel additive? 24.23.196.85 (talk) 04:38, 10 February 2013 (UTC)[reply]
Long time later but in retrospect petrol/gasoline would have been clearer. Nil Einne (talk) 21:57, 20 July 2013 (UTC)[reply]

Jupiter and the Romans[edit]

The ancient Romans gave Jupiter its name after their main god. Did they know that it was the biggest planet in the solar system, or is it just a coincidence? It's certainly not the brightest object, or even the brightests planet visible in the northern skies, so it's not immediately clear to me why they assigned it such importance unless they knew it was so large. 202.155.85.18 (talk) 07:47, 8 February 2013 (UTC)[reply]

It is usually the brightest planet in the night sky. (Venus is seen only before sunrise or after sunset.) Ruslik_Zero 09:08, 8 February 2013 (UTC)[reply]
I was taught at school that they believed that these objects in the heavens (asters) actually were gods, like the Sun, which is the most obvious godly thing. Many certainly must have thought that Jupiter-the-God and Jupiter-the-aster were the same thing or that one was a representation of the other, but wikipedia does not explain that relation very well, for example our article Jupiter (mythology) doesn't mention the words astronomy or astrology at all. As to why it became so important in the mythology is not related to its actual size (which they could not calculate very accurately), but to the stories in the myth. --Lgriot (talk) 09:21, 8 February 2013 (UTC)[reply]
No, there's no way they could have known. No planet is big enough to show a disk to the naked eye, so they couldn't have measured any planet's angular diameter.* They also couldn't have measured distance because the only way was by parallax, but even the parallax of Mars at closest approach is less than 1 arcminute from opposite sides of the Earth.
Not only is Jupiter usually the brightest planet, it's also brighter than every star. Venus is not always visible, even in the morning or evening; it's often hidden in the glare of the Sun, although the Romans would have known this instead of thinking it disappeared.
*Actually, there is one easy way to measure angular diameter. Hold your head absolutely still, and watch as a planet disappears behind a building due to Earth's rotation. A star would disappear instantly because its angular diameter is tiny, whereas planets take a few seconds. The longer it takes, the larger the planet's angular diameter. If you're patient enough, you can also use the Moon instead of a building (see lunar occultation). --140.180.247.198 (talk) 10:37, 8 February 2013 (UTC)[reply]
That will fail for a number of reasons, including atmospheric blurring, but most importantly the fact that it is nowhere near possible to hold your head still enough. Looie496 (talk) 16:49, 8 February 2013 (UTC)[reply]
You've obviously not seen A Clockwork Orange. μηδείς (talk) 17:33, 8 February 2013 (UTC)[reply]
Interesting... I feel like there must be a way. In theory, if you have a thin piece of wood with slits of gradually increasing width through it, you should be able to move your head, looking at the images through the piece of wood, until they stop getting brighter. I have no idea if the ancients did this (or, more likely, some much more clever version) Wnt (talk) 19:04, 8 February 2013 (UTC)[reply]
I don't understand the Clockwork Orange reference, but atmospheric blurring is nowhere near enough to make a star look like a planet. On a normal night, astronomical seeing produces a disk around 1 arcsecond across, whereas planets are tens of arcseconds across. As for holding your head still, you can use a sufficiently distant object--a few hundred meters is more than enough if you can hold still to within 1 cm--or you can use a vice to clamp your head. --140.180.247.198 (talk) 19:37, 8 February 2013 (UTC)[reply]
The "vice to the head" thing was the Clockwork Orange reference. The protagonist's head being held in a vice-like apparatus and being forced to watch images is one of the iconic scenes from the movie (if not from the book). See the image at A_Clockwork_Orange_(film)#Psychology. -- 205.175.124.30 (talk) 19:48, 8 February 2013 (UTC)[reply]
"It's a sin!" μηδείς (talk) 01:22, 9 February 2013 (UTC)[reply]

About isotopes[edit]

Do heavier isotopes of a same element always have

  1. heavier elements
  2. heavier compounds

than those of lighter isotopes?--Inspector (talk) 12:23, 8 February 2013 (UTC)[reply]

What do you mean by "heavier elements"? The answer is yes for that part, if I understand you correctly. By volume, they are heavier in their elemental state. The compounds would be heavier, unless they combine with lighter isotopes of other elements. Otherwise, yes, the compounds would be heavier, since (as I understand it) the properties of isotopes are more or less the same, other than molar mass and stability. IBE (talk) 13:33, 8 February 2013 (UTC)[reply]
We have a whole article about isotopes, covering the differences and similarities. DMacks (talk) 16:36, 8 February 2013 (UTC)[reply]

Check whether the following two points are right or wrong.
1. Sound energy is a form of energy, but sound is a mechanical wave (not energy). Light is neither energy nor a form of energy, but it is an electromagnetic radiation. Radiant energy is the energy of electromagnetic waves. Heat is not a form of energy, but heat itself is energy.
2. Energy of waves or radiations is directly proportional to frequency and inversely proportional to wavelength.
I don't know whatever I have written is right or wrong. Kindly correct the wrong statements. Sunny Singh (DAV) (talk) 12:43, 8 February 2013 (UTC)[reply]

Sound, light and heat are all forms of energy. You could probably do a bit of reading first, and post a couple of references to articles which didn't quite answer your questions. IBE (talk) 13:35, 8 February 2013 (UTC)[reply]
Not sure about this usage of "forms of energy". Energy is an attribute of certain physical phenomena and systems. Saying "sound, light and heat are all forms of energy" is like saying "my blue shirt, my red tie and my brown shoes are all forms of colour". Gandalf61 (talk) 13:47, 8 February 2013 (UTC)[reply]
The easiest way to think of energy is "the ability to cause a change". The important things about this ability is that it is a) quantifiable and b) conserved. That means that you can measure it, and that it doesn't go away, nor is it created (though it can become unusable, see entropy and second law of thermodynamics). The various "forms of energy" are the (somewhat arbitrary) way in which we organize or categorize the various kinds of changes. Like all categorization schemes, these aren't rigidly defined categories which are inherent in the system itself, but are human-created distinctions which allow us to extract more understanding about the system. So, we can talk about energy in terms of "kinetic" and "potential" forms (that is, broadly speaking, energy which is currently causing a change, and energy which is "stored up", waiting to cause a change at a future time); or we can talk about it in terms of the types of changes that occur (mechanical energy, chemical energy, light energy, heat energy, etc.) None of these definitions have bright line boundaries; they're all a little fuzzy around the edge, depending on the specific model you are using. It's important also, to note that energy is not a kind of stuff you can hold in a bottle. It's a way to quantify changes. The way in which energy, time, and matter work together is the basic definition of the science of physics: physics describes how every kind of change occurs through mathematical models. Wave energy is just energy which occurs in a repetitive pattern. Some waves are obvious (sound waves, ocean waves). There are some phenomena which can be modeled as either wave energy (oscillations of a field rather than oscillations of collections of particles) or as particles-in-motion; the fact that these phenomena could be modeled both ways is called wave-particle duality. Energy is a really complex concept, so I hope this sort of explanation is helpful. The iconoclastic physicist Richard Feynman, easily one of the most intelligent and articulate physicists of the last hundred years readily admitted that even he couldn't adequately understand nor define what energy really is. I hope that helps some. --Jayron32 14:26, 8 February 2013 (UTC)[reply]
We end up always defining something by its properties. Defining something by what it does (like energy is the "the ability to cause a change") seems to me to be the standard way of defining things. You define 'brown' by the wavelength that is reflected by a brown object. "Function" is "something that takes one value and outputs another. Knife is something that cuts, and can be hand-held. OsmanRF34 (talk) 15:47, 8 February 2013 (UTC)[reply]
What's the difference between "form of energy" and "energy"? If it's energy (like sound, light), it has to be in some form. If it's not a form of energy (like length) it cannot be energy. OsmanRF34 (talk) 14:23, 8 February 2013 (UTC)[reply]
Article heat starts with this "heat is energy transferred from one body to another by thermal interactions". This made me think that heat is energy but not a form of energy. Jayron you wrote heat energy, but this answer says writing heat energy is bad english. This is where the confusion starts. Make it clear. Sunny Singh (DAV) (talk) 17:02, 8 February 2013 (UTC)[reply]
Meh. Heat energy, thermal energy. Whatever. The language is a bit fuzzy in this regard; it depends entirely on which text book you're reading that day. Also, your sentence " heat is energy but not a form of energy" is actually logically silly: a form of something is merely an example or category of it. You can't simultaneously be something, and then not be the same thing. Heat (or thermal energy, if you prefer) is merely the energy of molecular motion. Some texts will draw the distinction between heat and thermal energy insofar as heat is the transfer or movement of that thermal energy. In that way, heat is to thermal energy as work is to mechanical energy. One very common expression of energy is the equation ΔE = q + w; that is ΔE (the total energy changes or transfers in a system) is equal to the sum of q (the heat) and w (the work). So heat just means "how thermal energy moves around" in the same way that work means "how objects move around"; and one way to think of energy changes in systems is to think of such changes as either heat or work or some combination thereof. --Jayron32 17:37, 8 February 2013 (UTC)[reply]
So, this means saying heat or heat energy; light or light energy; sound or sound energy, all are same. Sunny Singh (DAV) (talk) 05:14, 9 February 2013 (UTC)[reply]

.

Upon reading Sunny Singh's question here and last time (Nov 14 2012), and the answers, I think that folk have not percieved Sunny Singh's fundamental problem, though this time people have come close to one part of it. Sunny Singh has two areas of difficulty: a) he's not too good on the English language, and b) he doesn't have a clear idea of what energy is. To a certain extent we have to guess what Sunny wants of us, and what he needs of us, because his English is somewhat cryptic. It would be helpfull if Sunny responds to this posting and lets us know if I and the other posters are on track or off track.
First, some points on English: It is indeed poor English to say "heat energy", "light energy" and "sound energy", though you will find those phrases in lay writing and occaisonally even in professional text books. The reason why it is poor English is the same as it is poor English to write "cat animal", "goat animal", "snake animal" - becaue they ARE all animals - each phrase is much like writing "animal animal". The only difference is, everybody knows cats, goats, and snakes are all forms of animals, but those of us without a good science education may not know that heat, light, and sound are all forms of energy. Got the idea? Just as we write "cat" when we mean cat, and not "cat animal", we write "light" when we mean light.
Before answering the specific questions you posted, Sunny Singh, let's try and make what energy IS, very clear. OsmanRF gave you a key. Energy is that which can do something or change something. Energy is something that can be accumulated or converted - energy mathematically is the product of power and time (or if you like, intensity x time). For example: an electric lamp rated at 50 watts, when operated for 10 seconds, consumes 500 joules of electricity (another form of energy) and converts it into light and heat, both emitted to a total extent of 500 joules. Another example: A loudspeaker is fed from a stereo amplifier playing a pure tone at 10 watts for 1 minute. That loudspeaker has consumed 600 joules of electricity (from the amplifer) and converted it into a total of 600 joules of heat and sound. If the loudspeaker was 100% efficient, it would emit 600 joules of sound, and no heat. Get the idea? Energy is somthing that, over time, gets converted from one form to another.
Now, we can look at your posted questions, Sunny. As I said, your English is cryptic, so I've altered them a bit here and there to mean what I think you meant to say. Let us know if I guessed wrong on what you meant.
1a Sound is a form of energy - correct.
1b Sound is a mechanical wave, and a mechanical wave is not energy - This statement is wrong because a mechanical wave is a form of energy, because a transducer, such as a microphone, can convert it over time into another form (electricity).
1c Light is not energy - this statement is obviously wrong
1d Light is electromagnetic radiation - Yes it is, and electromagnetic radiation is another form of energy - by suitable means light can be converted over time into another form of energy - e.g., into heat by means of a black object.
1e Heat is not energy but heat itself is energy - This statement is wrong because heat is a form of energy - it can be converted into another form of energy (light, sound, etc).
2. The energy in waves or radiation is directly proportional to frequency and inversly proportional to wavelength. - This statement is completely wrong. I'm guessing Sunny included it because he was confused by reading physics texts while not having a clear understanding of energy first. For any sort of wave or radiation (i.e., mechanical waves such as sound, or electromagnetic radiation such as radio waves), the wave has magnitude, or intensity, measured in watts. Remember, energy is magnitude multiplied by time. Frequency and wavelength has NOTHING to do with it. If 2 watts of sound is coming out of a loudspeaker, then that loudspeaker is emitting 2 joules per second of energy, regardless of frequency. Confusion for lay people can come when discussing electromagnetic energy, which is a combination of electric and magnetic fields. Physicists don't realy know what electric and magnetic fields are. But we have long learnt that for many purposes the mathematics of waves gives the right answers, and for other purposes the mathematics of particles (photons) gives the right answers. As the theory of electromagnetic radiation is a dual of waves and photons, we can combine the two and say things like "photons are emitted at a frequency of x hertz". The higher the frequency the more energy in each photon. Note that saying "photons are emitted at 20 megahetz" (1 megahertz means 1 milion cycles per second) DOES NOT mean 20 million photons emitted per second. The higher the frequency, the fewer photons needed. The greater the power, the more photons are needed. Each photon is a packet of energy. The amount of energy in each packet is proportional to the frequency of the equivalent wave.
Wickwack 124.178.169.170 (talk) 10:04, 9 February 2013 (UTC)[reply]
Energy is an attribute of physical phenomena and systems. A thrown ball has various attributes, including height/altitude, velocity, kinetic energy and potential energy. We can say "the ball has energy" but it is incorrect to say "the ball is energy" or "the ball is a form of energy". This would be like saying "the ball is velocity". Similarly, a sound wave or an electromagnetic wave has a frequency, a wavelength, an amplitude and an energy level (or, more generally, it has a spectrum of these attributes). We can say "a wave has energy" but it is incorrect to say "a wave 'is energy" and also incorrect to say "a wave is a form of energy". Gandalf61 (talk) 11:57, 9 February 2013 (UTC)[reply]
No, that isn't right. You are correct in saying that it is wrong to say a ball is energy. It is clumsy to say that a ball of mass m at some height z is a packet of potential energy. But the analogy does not stretch to waves. A fundamental propery of waves/radiation is magnitude/intensity. The energy of a wave is merely the product of magnitude and time as I said. Any two and you can calculate the third. Thus the three things, magnitude, time, and energy, are not independent attributes. Wavelength can be calculated from frequency and velocity of propagation (which is determined by the medium) - so these are not independent of each other, either. However, frequency/wavelength/velocity IS independent of magnitude, energy and time. When a wave passes from one lossless medium to another, neither magnitude nor frequency is changed. Thus, if we want to boil waves/radiation down to the simplest set of fundamentals, there are two fundamentals: magnitude and frequency (not counting direction of propagation and polarisation). These two attributes completely describe the wave. These two attributes, along with identifying what sort of radiation it is, completely describe the radiation. So, it is perfectly ok to use the energy describing words "light", "sound" etc to mean energy in the respective energy form. There is no need nor sense in saying things like "the energy in the wave." One does not say "the radio transmission has an energy of 200 joules per second" (this is much the same as why it is bad English to say "sound energy", "light energy" etc); we can just say "the radio transmission IS 200 joules per second." (More normally we would say "a 200 Watt transmission"). It is entirely correct to say "sound is a form of energy", "a wave is a form of energy", etc. Wickwack 60.230.225.90 (talk) 13:23, 9 February 2013 (UTC)[reply]
Most of that is nonsense. Obviously a wave is not completely described by its magnitude and frequency alone. A wave has many other attributes, some of which you mention and then randomly discard. And you seem to be forgetting that most waves do not even have a single frequency. No-one says "the radio transmission IS 200 joules per second" or "the radio transmission is 200 Watts". The phrase "a 200 Watt transmission" is obviously short for "a transmission that has a power of 200 Watts". Saying "sound is a form of energy" makes as little sense as saying "my shirt is a form of colour" or "a car is a form of momentum". Sound energy is a form of energy, but the sound wave itself is neither energy nor a form of energy - see our article on forms of energy. Gandalf61 (talk) 15:57, 9 February 2013 (UTC)[reply]
I just love people who provide links to Wiki articles or web pages, but those articles do not support their argument - as is the case with Gandalf's post. You have not read or understood the word "fundamental" in my post above. As I said, the two fundametal properties of waves/radiation are magnitude and frequency. All the other parameters you mentioned, wavelength, amplitude, energy, and more, can be calculated from the two fundamental properties plus time and velocity. Velocity is not a property of the radiation, it is fixed by the medium. I haven't discarded anything. For example, let's say a source is emitting in vacuum a light bean of magnitude 200 W at a frequency of 6 x 1014 Hertz. The energy is 200 joules per sec. The velocity is the speed in vacuo, ~ 300 x 106 m/sec. Thus the wavelength is 300 nanometre. Now lets say the beam passes from vacuum into a block of perfect silica glass. In this glass the velocity is only about 150 x 106 m/sec (I've tweeaked the value a bit to make the meantal arithmetic easy). Magnitude is still 200 Watt. Frequency is still 6 x 1014 Hertz. Energy is unchanged. Wavelength is doubled. See how it works? See that magnitude and frequency are fundamental to the wave, and all the other attributes can be calculated? Yes, more than one frequency may be present. That does not invalidate what I said - it merely means there is more than one wave superimposed in the medium or space. The total amplitude can be calculated from the addition of the component waves. Yep, nobody says "the radio transmission is 200 joules per second", usually. It is perfectly valid to do so though, and sometimes, when the energy is of primary interest, we do just that. It is just more convenient in most situations to say "a 200 Watt transmission.
Now, what is different about saying "a car is a form of momentum", and "sound is a form of energy"? Lots. Momentum is is just one possible attribute of a car - a car has a vast mumber of independent attributes - height, number of doors, colour of paint, etc etc. A particular car may have at some point in time, a lot of momentum, a litle, or no momentum at all. Regardless, it's still a car. But sound is completely described with a couple of fundamentals. If the sound has zero energy, there is no sound.
Wickwack 121.215.80.105 (talk) 02:05, 10 February 2013 (UTC)[reply]
"Magnitude and frequency are fundamental to the wave, and all the other attributes can be calculated" ... more nonsense. For example, the phase of a wave cannot be calculated from its magnitude and frequency. Gandalf61 (talk) 05:45, 10 February 2013 (UTC)[reply]
True, sort of, but phase is not relevant to the question the OP asked. Phase is not an intrinsic property of a wave. You can only measure phase with respect to an independent timing reference. Keit 121.215.141.120 (talk) 05:56, 10 February 2013 (UTC)[reply]
Correct, Gandalf has not got a clear picture. If a light beam or radio beam in deep space passes a little green alien, he can say (translated into English and SI units) "I sense a light wave, frequency 6 x 1014 Hertz, magnitude 200 Watts." But he can assign to it any arbitary phase he likes, because the wave does not itself have a frame of reference. Of course phase cannot be calculated from just magnitude and frequency, as phase is not a wave property - it is a function of both the wave AND the separately existing timing reference. Wickwack 121.221.41.198 (talk) 06:38, 10 February 2013 (UTC)[reply]
Total nonsense. Of course phase is a property of a wave, and is just as "intrinsic" as its magnitude, frequency, wavelength etc. You cannot calculate the superposition of two waves from knowing only their magnitudes and frequencies. You also need to know the phase of one relative to the other. So the waves cannot be completely described by their magnitudes and frequencies alone. Your selection of magnitude and frequency as the only "fundamental" properties of a wave is both arbitrary and unscientific. Your "frame of reference" argument is nonsense because it leads you to the ridiculous conclusion that energy is not a property of a thrown ball because its potential energy is measured relative to an arbitrary ground level and its velocity (and hence its kinetic energy) is measured relative to an arbitrary frame of reference. Your whole chain of reasoning that you are making up to justify your incorrect statement that "sound is a form of energy" is flawed at every step. Gandalf61 (talk) 09:10, 10 February 2013 (UTC)[reply]
You are being silly now. If two waves are identical in frequency, you can determine a phase difference - really this is just using one as the frame of reference to the other. You cannot, without a third reference say one wave has this phase and the other wave has that phase. If two waves differ in frequency, the phase difference is constantly changing over time, and you still can't assign a phase to either one. Phase is NOT a property intrinsic to a wave. Nowhere have I ever said that energy is not an attribute of a ball, thrown or otherwise. It is one of a multitude of attributes of a ball. Go back and read more carefully. Wickwack 120.145.68.197 (talk) 09:55, 10 February 2013 (UTC)[reply]
But the logical conclusion of your own arguments is that energy is not an "intrinsic" or "fundamental" property of the ball since it is measured relative to an arbitrary frame of reference. And the same conclusion can be applied to the potential and kinetic energy of any other system - you cannot allow those to be "intrinsic" or "fundamental" properties either. Or chemical energy, which is also only ever measured as a relative difference, not an absolute quantity. I agree these are all silly conclusions - but they are the conclusions that result from your own flawed reasoning. By repeating your nonsense about "fundamental" properties and "sound is a form of energy" you appear to be trolling, so I am done here. Gandalf61 (talk) 11:53, 10 February 2013 (UTC)[reply]
No, that does not follow from my arguments. You have not recognised the difference: energy IS a property of balls and waves (over time), and does not require an external frame of reference, whereas phase does require an external frame of reference. The convention of asigning zero chemical energy to common simple chemical forms (e.g., O2, N2, C, etc), and relating other substances back to them is a matter of sensible convenience and is somewhat arbitary. Wickwack 121.215.66.37 (talk) 14:19, 10 February 2013 (UTC)[reply]
Both Wickwack and Gandalf61 are missing the importance of understanding and communicating context. Analogous to forms of energy might be forms of liquids, i.e. water. Water is not always liquid, but it is a liquid when used in context. To say water is a form of liquid is to say water has the property of a liquid. If X is a liquid then water is an instance of X. Same with forms of energy. Sound, light and heat are each forms of energy. Each are distinguished of course by other properties but they nevertheless each have the obvious property of having energy. Contrary to Wickwack's Bad English nonsense, we sometimes do need to make it known which property we are talking about. For instance we don't say "light" equals mc squared, but we might say that radiant energy does. In nontechnical usage, heat means simply the quality of being hot which merely correlates with heat energy. Heat is energy, but only when it is understood that energy is what is being talked about, because context matters... and when its lacking we often use terms like "heat energy" (per these sources) to make it clear when its possible. --Modocc (talk) 13:49, 10 February 2013 (UTC)[reply]
Yes, we do sometimes need to make it plain, especially if the words are intended for lay people - I believe I said so myself. That doesn't make what I said nonsense though. I was trying to address what I think is the OP's difficulty, which seems to be not a clear idea of what energy is. Yes, you and I agree that Sound, light, and heat are forms of energy - soemthing crucial to the OP's question. For some reason Gandalf does not agree, and he's wrong. Wickwack 121.215.66.37 (talk) 14:12, 10 February 2013 (UTC)[reply]
Modocc - if you are equating "is a form of energy" with "has the property of having energy" then is it correct to day "a thrown ball is a form of energy", "gunpowder is a form of energy" or "this book is a form of energy" ? If not, then can you explain why not (if possible, without resorting to Wickwack's mystical and debunked notion of "fundamental properties") ? Gandalf61 (talk) 16:26, 10 February 2013 (UTC)[reply]
In the broadest possible terms, with current physics and AFAIK, there does not exist any physical object which does not have mass-energy. Indeed, the molecules of every object has mass-energy. Thus, each of your statements are actually true. In fact, I recently burned some moldy books in a bonfire. Now those books I had are nonexistent (in the present), but their energy changed form though due to the law of conservation of energy. -Modocc (talk) 17:59, 10 February 2013 (UTC)[reply]
Okay, so with your usage every physical entity - sound waves, radiation, chemical substances, material objects etc. - is a form of energy. That is a consistent usage (unlike Wickwack's nonsense) but it is different from the definition of forms of energy in our article. Gandalf61 (talk) 10:36, 11 February 2013 (UTC)[reply]
The article lists mass as a form of energy along with Einstein's mass-energy equivalence E = mc2 which defines the energy content of matter. -Modocc (talk) 12:18, 11 February 2013 (UTC)[reply]
I must say that the above debate is very dissapointing. If the OP has long since given up I shouldn't be surprised. The talk about the E = mc2 mass -energy equivalence, while not wrong, is hardly relevant to the OP's difficulty - it is just a distraction. Gandalf61 is, at best, very confused, and at worst, just a troll. Wickwack says sound, light, waves etc are forms of energy. This is consistent with reality, and agrees with the Wiki article cited more than once by Gandalf61. However, for reasons unfathomable, Gandalf61 claimed (last sentence in his 2nd post and elsewhere) that they are not. In that he is talking nonsence. Yet, in his last post above, he himself used the term "forms of energy". Looking for a way to rationalise Gandalf61's posts, about the only think I can think of in his claims about phase etc versus Wickwack's fundamentals, is that Gandalf61 does not understand the meaning of the word "fundamental".
Giving Gandalf61 the benefit of the doubt, i.e., assuming he is not trolling, and because some folk do think it means something different (simplicity sometimes), here is the directionary meaning of "fundamental": basal, serving as foundation, primary, an essential. In scientific and engineering work fundamental means that from which everything else relevant can be inferred or calculated. So, Wickwack's assignment of frequency and magnitude as fundamental attributes is correct. As he said, all other attributes of waves (eg energy, period) can be calculated from these alone. In fact that is basic stuff familiar to any Engineer. It is no different to describing a circle. A circle can be defined entirely by just it's radius. A circle does have other attributes - diameter, circumference, area. But these can be calculated from the radius. For a circle, you can assign any one of radius, diameter, circumference, area as a fundamental - the only one that you need quote to define a circle. In the same way, all you need to define a wave is 2 things: frequency and magnitude, though you can just as validly use another two, e.g., period and magnitude. Standard physics textbooks, eg Physics for Scientists and Engineers, Giaconelli (in 3rd edition, see Chpater 15 Wave Motion, especially 15-1 thru 15-4), say just the same thing.
As Wickwack said, phase is not an attribute of a wave. Without an external reference, phase has no meaning - I would have thought that was obvious.
Gandalf61's repeated discussion of balls has no value. Quite clearly, frequency and magnitude completely specify a wave (if the type of wave is known of course, eg sound, light, radio, etc), and if you have finite values for frequency and magnitude, you have a wave, and you have energy being transported. If you don't, there is no wave, and no energy transport. Clearly, waves are thus a form of energy - sound is a form of energy, light is a form of energy, etc. Balls are quite different. While you can completely specify sound with just the two fundamentals of wave energy (frequency and magnitude), you cannot specify a ball only by quoting its energy. As Wickwack said, take away (all the kinetic and potential, thermal, etc) energy, and you still have a ball. Take away the energy from a wave, and there is no wave. Balls have lots of other attributes - shape, size, mass, structure, etc. That's why you can't just say a ball is a form of energy, unless there is special context. Outside that context, it isn't. A ball is an object having size, shape, mass, etc, and sometimes some energy. It is quite silly of Gandalf61 to try and consider balls and waves as equivalent, in the context of the OP's question.
Ratbone 120.145.203.206 (talk) 15:43, 11 February 2013 (UTC)[reply]
Gandalf61 does understand that energy is a property of matter such as balls, although he was uncertain and mistaken about the semantics of "forms of energy". Wickwack did a fairly good job of critiquing the OP's statements as requested, so I hope the OP is satisfied with his answer, but Wickwack's didactic pedantry about "sound energy" being poor English, even though we have an article titled "Sound energy" which is listed in the "Forms of energy" article was distracting too. Sure take away the intensity of a sound's energy and there is no sound, but that's equivalent to taking away a piano's mass, in which case you have no piano nor music. -Modocc (talk) 19:59, 11 February 2013 (UTC)[reply]
That seems rather contrived. Take away the mass of anything that has mass, and in practice you haven't got it anymore. A better analogy to Gandalf's arguments is take away the musical sounds, say by playing it in a vacuum. You've still got a piano. We don't actually write, normally, "sound energy", "light energy", "radio wave energy", etc. We just write "sound", "light", "radio wave". But we can't just write "ball" and expect people to think of it as energy. We can't just write "piano" and expect people to assume it is being played. Re-using one of Wickwack's analogies, we don't write "cat animal" because we know all cats are animals. You can't have a cat that is not an animal. There's nothing in a cat that isn't animal (well, maybe the oddd bit of grass in its' tummy that it ate because it felt ill). You can't have a piano without mass. But we do write things like "the black cat", because cats come in different colours, and we wanted to say what colour that particular cat is. We say "upright piano", or "grand piano" if we want to state what kind we mean. There's no need to say "sound energy" because there is no sound without energy, and (mechanical) energy is all sound is.
A piano played in a vacuum is rather pointless, for you will still have music from the vibrant sounds transmitted from the strings to within its boards, for sound requires a medium. The energy of any sound, at its most basic level, is simply just a select fraction of the kinetic and potential energies of the particles of the medium. Moreover, mass is not just any arbitrary attribute either, its energy. Therefore taking away sound energy (or sound if you insist) is taking away mass anyway. As for the assertion that we automatically think of waves in terms of energy (which people clearly don't always do, for I'm quite fond of body-surfing them) and that we don't think of warm-blooded cats as energy, well... its probably because we don't usually need to and cats are a tad more complicated than a waveguide. -Modocc (talk) 05:10, 12 February 2013 (UTC)[reply]
Like Ratbone, I don't think this "mass is energy", presumably your E = mc2 line, helps the OP either. And mass isn't energy, it requires conversion into energy, by some process such as nuclear fission. Apart from that, thank you: yes, cats are more complicated - that was exactly my point. If you wanted to describe a cat to an alien from the next galaxy, using some dot points, you probably would not mention its' kinetic or potential energy. Just had a horrible thought though - with enough cats, which do have both mass and elasticity, you could have a wave passing through them. Agreed, many people do not think of waves as energy - but only because they are ignorant in science. Many people, including highly intelligent people, think that the universe and everything in it, substantially in the form that it is today, was created in 6 days, a few thousand years ago. That does not make it right. Those of us with at least a half baked education in science, and those of us who are engineers, should know that energy, that is, an interchange of some kinetic and potential (or elastic) energy as you said yourself (for mechanical waves) or the equivalent in electromagnetic waves, is what waves are. Wickwack 124.182.165.35 (talk) 07:25, 12 February 2013 (UTC)[reply]
Wickwack - I love the bit where you say "mass isn't energy" and then go on to belittle people who are "ignorant in science". What a clown. This thread just gets better and better. Gandalf61 (talk) 09:55, 12 February 2013 (UTC)[reply]
Is that the best you can do? No reasoned argument eh? You must be at last beginning to sense I must be right then. Wickwack 120.145.170.52 (talk) 11:01, 12 February 2013 (UTC)[reply]
Per mass-energy equivalence and conservation of energy mass is most certainly energy, no "conversion" between them is needed. For instance, one doesn't need to convert the energy of gluons to obtain the proton's rest mass or its total energy. The possibility of particle masses being wave energy was recognized by physicists such as Max Abraham before Einstein actually. I will add that in spite of the success of these principles, physicists are often either unclear or uncertain as to why the equivalence between these basic forms of energy exists, which is part of the reason why they continue to pour whopping amounts money into continued research. -Modocc (talk) 13:31, 12 February 2013 (UTC)[reply]
The first article you linked is about the E = mc2 equality. It's not about saying that mass is the same as energy, it about saying that if you convert a mass (the article gives nuclear fission as an example, but other methods of assembling and dissassembling particles are always conversions), then this is how much energy you can convert the mass into. It's about conversion. The second article you linked doesn't alter that, it just says the the convservation of energy principle had to be reviewed in light of the discovery that it can be converted to mass and back again. It's about conversion. However, for some theorists, a packet of energy (suitably sized and packaged up, not with brown paper but perhaps with some "string" http://en.wikipedia.org/wiki/String_theory) can manifest itself as a mass. Regardless, how does all this talk of mass-energy equivalence, gluons, and whatnot help the OP? How is it relavent to his questions about whether sound, light, and heat are forms of energy? How does it help him understand whether or not the energy in a radiation is related to frequency? Seems to me it has no relevance and may be for the OP a red herring. When one calculates how much energy is in a given sound wave, E = mc2 doesn't come into it. When you calculate the power of a light beam or radio wave, E= mc2 doesn't come into it. Wickwack 60.230.208.11 (talk) 14:51, 12 February 2013 (UTC)[reply]
From the first article I linked to: "According to the theory of relativity, mass and energy as commonly understood, are two names for the same thing, and neither one is changed nor transformed into the other." Apparently you are definitely not reading these articles carefully (Disclaimer: I didn't write these articles and its best to research the citations for a better understanding, and I've done my own reading of books and papers over the years so I do know what I'm talking about), and you could bring this up on the articles' talk pages and provide wp:RS for your view, which, in principle, dogmatically pigeon holes mass and energy into distinct categories. -Modocc (talk) 15:14, 12 February 2013 (UTC)[reply]
With the aid of Edit|find... sure enough, "According to the theory....: is right there under the heading Conservation of Mass and Energy, and is the "packet of" idea I mentioned (the article uses the phrase "a mobile form of mass) - I wasn't as dogmatic as you claimed. But you haven't answered how does this help the OP? How is it relevant to the OP's questions? Wickwack 60.230.208.11 (talk) 15:35, 12 February 2013 (UTC)[reply]
Of course masses are mobile and to be somewhat clearer, these photon packets are packets of mass-energy. As to why this helps the OP, it should be obvious, because the question's title is "Forms of energy", which could be rewritten without any loss of meaning as "Forms of mass-energy". -Modocc (talk) 15:50, 12 February 2013 (UTC)[reply]

Iranian and Iraqis anti-aircraft air defense[edit]

How does the present Iranian anti-aircraft air defense compares to the Iraqi anti-aircraft air defense at the beginning of the 2nd Gulf war? OsmanRF34 (talk) 14:14, 8 February 2013 (UTC)[reply]

I couldn't find any direct comparison, but this article is rather scathing of Iran's air-defence capabilities; "Iran has even more problems with its land-based surface-to-air missiles. Its only modern systems are short-range man-portable systems and some 30 short-range Russian TOR-Ms suitable only for point defense. Its other systems are 30 short-range Rapier fire units and 15 Tigercats of uncertain operational status. Its longer-range systems include roughly (154) U.S. IHawks, (45) Russian SA-2s, (10) SA-5s and a limited number of CSA-1 Chinese versions of the SA-2. All are obsolete."
Iraq in 1991 had spent a large amount of money on the best system that the Soviets (and the French to an extent) were willing to supply them with. By 2003, they only had whatever hadn't been destroyed in 1991. Although US Air Force general John W Rosa told journalists: "The Iraqi air defence system is one of the toughest, most complex systems that we see in the world... It's very capable. They're constantly working to improve it, and they have been.", Andrew Brookes, an air specialist at the International Institute for Strategic Studies in London said "It's rubbish".[8] Alansplodge (talk) 17:13, 8 February 2013 (UTC)[reply]
If Iran's anti-aircraft is as sophisticated as their new fighter jet,[9] then it may have come from the same oversized G.I. Joe kit. ←Baseball Bugs What's up, Doc? carrots→ 07:30, 10 February 2013 (UTC)[reply]

Kite with extremely long string[edit]

If someone had a kite with a very, very long string attached, how high could the kite reach before some physical process prevented it getting any higher? --Dweller (talk) 14:41, 8 February 2013 (UTC)[reply]

The current record may be 13,600 feet [10] but I don't think any physical process limits it to that low. "Go fly a kite" and try to better the record yourself? In the U.S. you will need FAA clearance though. Rmhermen (talk) 15:25, 8 February 2013 (UTC)[reply]
At such altitudes, does it still work like a kite? bamse (talk) 16:58, 8 February 2013 (UTC)[reply]
Yes, but at some altitude there won't be enough air to create enough lift (airfoil effect) to overcome the weight of the string. 74.60.29.141 (talk) 17:05, 8 February 2013 (UTC)[reply]
The limit has to be the weight of the kite and the weight of the string versus the amount of lift it can generate. As the kite gets higher, the string gets longer - and therefore heavier. You might think that this means that you need a larger kite to lift the weight - but that increased area also increases the forces on the string because the drag force on the kite is proportional to the area. So a larger kite needs a stronger (and therefore heavier) string...and a bigger kite to lift that additional weight...and so forth.
The difficulty is that if you double the area of the kite, you double the amount of lift (and drag) that it has. You also double its weight and you have twice the tensile strength needed in the string - which demands a string with twice the cross-sectional area - which has twice the weight per unit length. Hence, making a kite larger doesn't increase the height it can fly at.
To solve that, you're going to need to consider varying the thickness of the string along its length. Near the kite, the string has to be strong enough to support it's entire weight - plus the drag forces on the kite. But close to the ground, it only has to be strong enough to counteract the drag force. So you could save weight by using a thicker cable up near the kite and less thick close to the ground...but there must be a limit to how much that helps.
Ultimately, you're going to start to find that the lessening of the density of the atmosphere would reduce the amount of lift that the kite can get...but it would also reduce the drag on it, allowing for a larger kite without needed a heavier string.
Arguably, the Space elevator idea is the ultimate kite-like thing - it would not be kept up there by the wind, but by the centrifugal force as the earth rotates. Issues of tether strength and the variation in thickness over altitude are a big question for that kind of structure.
SteveBaker (talk) 17:09, 8 February 2013 (UTC)[reply]
[(edit conflict)] ~ However, there are aerodynamic effects on the string itself that can provide lift, as per ballooning spiders -which is still not well understood.   I haven't checked this out yet, but you might want to read:  Aerodynamics of Kites.     74.60.29.141 (talk) 17:17, 8 February 2013 (UTC)[reply]
In a place with air flowing quickly upward, like the a stationary warm-core storm, there should be enough lift on the string to overcome it's weight. StuRat (talk) 18:41, 8 February 2013 (UTC)[reply]
Gliders, hang gliders and paragliders are like kites without string. A very lucky paragilder reached 32,600 feet due to cloud suck and lived to tell the tale. World altitude record for hang gliding is 38,800 feet, although this was a balloon launch not an ascent from ground level. World altitude record for gliding is 50,699 feet. Gandalf61 (talk) 17:22, 8 February 2013 (UTC)[reply]
The problem with gliders and such is that once they are high enough to get above the beneficial "slope lift" effects - they are entirely reliant on upwelling air to get their altitude. Significantly, once they are high enough, the effect of the wind is zero - the speed of the glider relative to the air is all that matters.

A kite, on the other hand, is tethered to the ground - so it can use the speed of the wind to gain altitude. It's speed relative to the air is whatever the wind speed is. But it's ability to make use of upwelling air is limited for the exact same reason...if it doesn't happen to be in an upward current, there is nothing that can easily be done to fix that. So, in principle, a kite can do better than a glider...especially if there is abundant wind and an updraft that happens to be where the kite is situated...but eventually, the weight of the string outweighs that advantage. SteveBaker (talk) 20:03, 8 February 2013 (UTC)[reply]

So, any idea of a reasonable maximum altitude? --Dweller (talk) 20:06, 9 February 2013 (UTC)[reply]
That depends on the string's strength-to-weight ratio more than anything else. 24.23.196.85 (talk) 20:14, 9 February 2013 (UTC)[reply]
For two kites then, one with typical commercial strength-to-weight ratio, and one superduper one designed by NASA engineers in their spare time. --Dweller (talk) 20:17, 9 February 2013 (UTC)[reply]
What material should I use for the string in each of the two cases? 24.23.196.85 (talk) 20:19, 9 February 2013 (UTC)[reply]
Just ran some calcs for your second case (the "superduper one designed by NASA engineers in their spare time"): with a string made of Kevlar (assuming that the string has a constant diameter, and that I didn't screw up anywhere in my calcs, which I might well have), the ballpark figure for the theoretical "highest height" would be over 80,000 feet! Yes, you read it right -- assuming my calcs are correct, you can literally "send it soaring up through the stratosphere, up where the air is clear"! Of course, you'd first have to get permission from the FAA, because this would be a major aviation hazard... 24.23.196.85 (talk) 21:04, 9 February 2013 (UTC)[reply]
The FAA doesn't have much authority over Dunstable Downs (see parag 4), but I'm sure that someone would be annoyed if I deployed an 80,000 ft vertical string without notifying them. Not least the airfield that lies immediately below the popular kite-flying venue! --Dweller (talk) 08:48, 10 February 2013 (UTC)[reply]
Actually, if you seriously want to fly a kite to Flight Level 800, then Alert, Nunavut is the place where you want to go. 24.23.196.85 (talk) 01:21, 11 February 2013 (UTC)[reply]

So, 80,000 feet seems to be the max, determined by the weight of the string and the amount of lift still in the atmosphere? That's a pretty long piece of string. Thanks guys. --Dweller (talk) 08:48, 10 February 2013 (UTC)[reply]

Yes, and it would cost you a lot more money than a "twopence for paper and string" (not to mention that the kite, too, would have to be made of synthetic fabric in order to survive the strong winds and turbulence in the upper atmosphere); also, even with a Kevlar string that's only 1 sq. mm. in cross-section, 80,000 feet of it would weigh over 75 pounds. (Not to mention all the other logistical challenges that such a record attempt would entail.) 24.23.196.85 (talk) 01:26, 11 February 2013 (UTC)[reply]
It's a lot worse than that. Unless the string is dead vertical and there's no wind loading (and if there's no wind you can't fly the kite), you get a catenary loading effect in the string. Visualise an inelastic string that is slightly curved due to gravity, wind loading, or both. If the centre of the string length is displaced x metres, the ends are drawn towards each other in order to keep the same string length. The distance each end is drawn in is very much smaller than x and there is in consequence a considerable force mulitiplication as with any lever where the load moves a larger distance than the lever end. So, while the string may weigh only 75 pounds, the pull on the kite will be much much larger. In practice this will force you to use a much longer string so it can curve considerably - this will directly increase the weight of the string. Wickwack 121.221.38.48 (talk) 11:18, 12 February 2013 (UTC)[reply]
Darn, I knew I missed something... 24.23.196.85 (talk) 03:15, 13 February 2013 (UTC)[reply]
If there were a number of kites along the string as in the picture on the right
then one needn't worry about the weight, and the air going in different directions at different heights might actually help by reducing the horizontal force as well. Don't know if anyone has tried that though. Dmcq (talk) 14:00, 12 February 2013 (UTC)[reply]

Symbol of mass number[edit]

The symbol of atomic number is Z and the reason for this is beautifully mentioned in the article. The symbol of mass number is A. What is the reason behind using A as the symbol for mass number ? Show your knowledge (talk) 18:08, 8 February 2013 (UTC)[reply]

From google I get the drift that it stands for "Atomic mass number". ←Baseball Bugs What's up, Doc? carrots→ 22:15, 8 February 2013 (UTC)[reply]

Why don't the electrons fall into the nucleus?[edit]

Electrons are negatively charged and nucleus is positively charged, we also know, unlike charges attract each other. Why don't electrons come and stick to the nucleus of the atom ? Want to be Einstein (talk) 18:17, 8 February 2013 (UTC)[reply]

They are in orbit. Just as the Earth would fall into the Sun, due to gravity, but doesn't, because of it's momentum, the same is true of electrons. There is nothing to slow them down, so they just keep on going. When a stray electron "gets lucky" and slams into a nucleus, bad things happen, like it hitting a proton and fusing into a neutron. However, the vast amount of empty space between the electrons and nucleus, compared with the size of electrons, makes such collisions extremely rare, a bit less so where the electrons are moving faster, like inside giant stars. StuRat (talk) 18:55, 8 February 2013 (UTC)[reply]
I remember two things that I have read somewhere on Wikipedia but I am not able to find them: First, electrons don't orbit the nucleus unlike planets around sun. Electrons have random motion instead of well-defined circular motion. Second, the reason StuRat mentioned is not the correct answer to my question. Want to be Einstein (talk) 19:13, 8 February 2013 (UTC)[reply]
Here you get into the classical model versus quantum mechanics. I gave you the classical model. Look at Jayron's answer below for more of the quantum mechanics model. StuRat (talk) 19:24, 8 February 2013 (UTC)[reply]
Actually, your classic model doesn't work for the following reason: An electron is charged whereas the Earth is essentially neutral. Electrical charges emit radiation when the accelerate (under classical physics). See Larmor formula. An orbit is an acceleration (all turns are accelerations); so an electrically charged particle should be radiating energy constantly as it turns its orbit around the nucleus, this loss of energy should cause the electron to spiral into the nucleus. The reason the earth doesn't do this around the sun is that the earth is electrically neutral, and so does not radiate when it is accelerated, and so can maintain a steady-state orbit. The fact that an electron has an electrical charge means that if it really was a particle in an orbit, it would either need a constant resupply of energy or it would spiral into the nucleus. The fact that this doesn't happen may be one of the key impetuses for developing quantum mechanics to explain how the atom was able to maintain a steady state despite this. --Jayron32 19:34, 8 February 2013 (UTC)[reply]
Does a single electron radiate energy when it turns, as when deflected in a CRT screen ? Also, the quantum mechanics explanation seems to come down to "we don't why, just accept it", as you put it below, so that's not really any better, just more complicated. StuRat (talk) 19:38, 8 February 2013 (UTC)[reply]
I think that is the principle behind a syncotron. 202.158.103.42 (talk) 14:28, 9 February 2013 (UTC)[reply]
Did you mean a synchrotron? 24.23.196.85 (talk) 01:28, 11 February 2013 (UTC)[reply]
Yes, electrons radiate energy when they are accelerated; when the electron fired in a cathode ray tube hits the phosphor, it will do so with slightly less energy than it would be predicted if the electron weren't charged. That's a principle which has been known since almost before people even knew that electrons existed. The Larmor principle and the first description of the electron date to the same year (1897). However, the better way to understand the principle is radio. Radio waves (photons, or light energy) is generated fluctuating electrical current. The source of the radio waves is the energy given off by variations in electrical current; you speed up and slow down electrons, and they shed radiation as a result. By controlling how you vary the current, you control how the radio waves are varied, and you can transmit information that way. That works exactly because accelerating electrons give off radiation. If the electron in an atom were in orbit, it would be under a constant acceleration, and would be similarly radiating. That it isn't is why we can safely say that the electron isn't actually orbiting. --Jayron32 19:48, 8 February 2013 (UTC)[reply]
Oh, and the "just accept it" thing isn't an admonishment to claim that QM can't be understood; it's that the implications of QM cannot be visualized. The problem is that people want to have a picture, some analogue they could create with shapes and objects they are familiar working with. You simply cannot visualize an atom like this; the problems with accepting QM principles is that you have to abandon the need to have a visual model which can represent them. So, I'm not saying "just accept it" to mean "humans cannot understand this", I'm saying "just accept it" to say "there's nothing in the way that classical objects work that can be a good analogue of this". Since, as humans, our entire sensory experience is with classical objects, there is literally no convenient way to describe a picture which correctly displays QM principles. So you have to accept what the calculations and laws tell you without having a picture to go along with them. --Jayron32 19:54, 8 February 2013 (UTC)[reply]
So how does QM explain the underlying reason why a cloud with no moving parts has momentum ? Does it come down to anything more than "that's what we observe" ? StuRat (talk) 20:02, 8 February 2013 (UTC)[reply]
That's the point. Concepts like "clouds", "moving", and "parts" are defined classically. When you say it's better to picture an electron as a "cloud", that's still depending on drawing an analogue to a classical object, and atomic level physics simply doesn't have those analogues. Electrons aren't balls, and they aren't clouds. The cloud visualization is better (because it captures the idea of nonlocalizability) but it has its own major fault in that such a steady state cloud cannot also have momentum, at least if your depending on what your experience tells you a cloud is. Quantum scale physics has all of these problems in meshing with human experience. The entire concept of wave-particle duality is a completely different sort of problem, but it's of the same ilk. A wave is a type of movement. A particle is a thing. So how can a thing be a type of movement also? One particularly bad explanation of wave-particle duality is that light, for example, chooses which set of properties it has based on the application. That's wrong headed. Light behaves the exact same way all the time in all situations. We can construct situations where modeling light as a particle works better, and we can also construct situations where modeling light as a wave works better; but neither model really captures what light is. There is nothing your senses experience that allow you to be able to visualize what light is. The best we can do is fudge together some classical concepts like "waves" and "particles": it's a shortcoming of human perception that's the problem. What we observe is a set of properties: the ways in which atoms interact with each other, the ways in which molecules form and behave, the ways in which electrons around a nucleus absorb and emit energy, etc. We run experiments and get data from those experiments. If we try to match that data with the predictions made by the equations of classical physics, it just doesn't work. That's what quantum physics does for us: it gives us a new set of equations that correctly match the experimental data of how electrons work and also correctly match the experimental data for everything else as well. As I mention below, the reason we keep the old equations around for the non-atomic stuff is that they're more convenient to work with, not because the quantum equations don't also work. --Jayron32 20:18, 8 February 2013 (UTC)[reply]
But don't you see a problem with just saying "that's the way it behave because that's what the equations say" ? This comes up in string theory, where we can come up with any of several set of equations, all of which match observations to the same degree. So, which is right ? Are we just guessing here ? StuRat (talk) 21:17, 8 February 2013 (UTC)[reply]
You're starting to edge up against a complete misunderstanding of what science is, Stu. Science doesn't tell us what is right, science tells us what works. The fact that there are multiple solutions that produce the same result is why things like string theory are open areas of research and exploration in science. We don't know the answers to the unanswered questions in physics, and a big part of that comes down to working out which of any of a number of competing theories (if any of them so far proposed) is more useful in explaining the current inadequacies in our existing models. "All models are wrong, some are useful" is an important aphorism here in understanding this. We're not trying to decide absolute rightness, we're trying to come up with more and more accurate models; but no model will ever be complete. The ones we have now are fantastically useful and accurate, much more so than Newtonian physics we used to work with (and still do work with where Newtonian physics agrees with the better, but more complex and esoteric, models). As simply as I can say it is this: the more modern theories (like QM and GR) which have gained broad acceptance are better than classical physics because the explain more phenomena in better detail, and match observed data more accurately than classical physics does. There are areas of open exploration in science (like the String Theory you keep bringing up) which are attempt to add another layer of accuracy and precision to our understanding, but which are not yet fully fleshed out nor fully accepted, because of the problems of the "several sets of equations which all match observations to the same degree". It is not a simple issue, there's thousands of physicists right now that are exploring those avenues and trying to resolve those problems you note. Most of them will go down dead ends, but perhaps someone working now will be able to add some positive confirmation to something like string theory, or another unifying theory, or maybe come up with something else. Unlike things like quantum mechanics and general relativity, these other ideas are just too new and too untested to produce anything like universal acceptance. One thing that is for certain, however, is that the new models will still need to agree with observable data at all scales and for all phenomena; if they don't they're not very useful. If they do, but don't add to the corpus of explained phenomena, then they're also not very useful. Useful new models are only those which match everything we already know AND which help to explain something we can't already explain. --Jayron32 22:35, 8 February 2013 (UTC)[reply]
When QM was new, it didn't predict everything, like the behavior of gravity, which was better explained by the older GR. Yet QRQM was generally accepted, because it explained other things better. StuRat (talk) 22:58, 8 February 2013 (UTC)[reply]
Assuming you mean GR in your last sentence, YES. --Jayron32 23:26, 8 February 2013 (UTC)[reply]
Nope, I meant QM. StuRat (talk) 00:07, 9 February 2013 (UTC)[reply]
Then the answer is still YES. --Jayron32 00:17, 9 February 2013 (UTC)[reply]
Agreed, thus refuting your statement that "Useful new models are only those which match everything we already know AND which help to explain something we can't already explain". StuRat (talk) 00:21, 9 February 2013 (UTC)[reply]
No, both Quantum Mechanics and General Relativity match everything we already know from classical physics (that is, neither of them contradicts the experimental results that classical mechanics also matches), but both theories also correctly predict things that classical physics gets wrong. That's why they are useful: They expand upon and replace simpler theories, and get more correct than the simpler theories do. If, for example, Quantum mechanics only gave us the exact same results as classical physics, and didn't give us better explanations for the things classical mechanics got wrong, it wouldn't be useful. If quantum mechanics could better explain some things that classical mechanics got wrong, but it was also wrong where classical mechanics was correct, it ALSO wouldn't be useful. That they agree in areas where observation confirms that classical mechanics was correct, and that quantum mechanics ALSO adds entire new explanations for physical phenomena that classical mechanics gets wrong (like the aformentioned problem of the electron that started this thread) is why QM is a generally accepted theory. It agrees with all of the observations classical mechanics agreed with, and adds a whole new set of observations that classical mechanics couldn't correctly predict. You can replace the words "quantum mechanics" with "general relativity" and the same statements hold true. --Jayron32 00:31, 9 February 2013 (UTC)[reply]
I wasn't talking about classical mechanics here. I was refuting your statement that "Useful new models are only those which match everything we already know AND which help to explain something we can't already explain" by giving the example where QM was a useful new model, despite not matching everything we already knew (from GR). Also, a model which explains nothing new, but is simpler, may also be useful. This comes up in reverse with string theories which want to add more and more dimensions, with little benefit. StuRat (talk) 00:21, 9 February 2013 (UTC)[reply]
The fact that GR and QM don't play well together is already well established and not in dispute. I already cited it as one of the great unanswered questions in physics. So, you're not adding anything surprising to the physics canon by noting that. --Jayron32 00:49, 9 February 2013 (UTC)[reply]
And I'm not trying to do that, am I ? I just used it to refute your statement, as I've said twice now. StuRat (talk) 06:43, 9 February 2013 (UTC)[reply]
The answer you gave is like explaining that the Earth is flat and stars are holes in a black cloth - that would be the "classical model". Your answer has been known to be incorrect for almost a century. 88.112.41.6 (talk) 19:39, 8 February 2013 (UTC)[reply]
It's not that simple. Quantum mechanics seems to work better on a small scale, and the classical mechanics approximation of relativity or relativity itself are better on a large scale. How to mesh them together is a problem that still confounds us. Things like electrons are right at the cusp of the two models, sometimes behaving like a probability cloud, and sometimes like a particle. So, relativity (and the classical mechanics approximation of relativity) isn't strictly right or wrong, and the same is true of quantum mechanics. StuRat (talk) 19:44, 8 February 2013 (UTC)[reply]
Your explanation is known to be incorrect. It violates basic laws of physics (conservation of energy and momentum). You may want to consider if your frequent contributions to the reference desk would be more valuable if you could recognize and admit when you give a wrong answer. This is not the first time I see you doing this. 88.112.41.6 (talk) 19:50, 8 February 2013 (UTC)[reply]
Actually, Stu, we do understand perfectly well how QM and classical mechanics mesh. QM laws reduce just fine to classical laws in the limits of measurements; they scale perfectly. There is no magical moment when an object stops obeying quantum rules and starts obeying classical rules; there's a continuum where the difference between the predictions made by classical physics and quantum physics decreases as the scales increase. That is, as size scales become larger, the quantum laws predict the exact same properties as the classical laws do. The entire universe can be accurately modeled entirely using quantum mechanics. We keep classical physics around because the math of quantum systems is a bitch, and if we can get the same results with the easier math, then why not? But it simply isn't true at all that quantum mechanics doesn't work on large scales. Only the converse is true: Classical physics doesn't work on small scales, but quantum physics works on all scales. That's why it's a better overall theory. --Jayron32 20:00, 8 February 2013 (UTC)[reply]
I don't agree that QM works on all scales. Here's a source which backs me up: [11]. And the opening sentence in our article says "Quantum mechanics (QM – also known as quantum physics, or quantum theory) is a branch of physics dealing with physical phenomena at microscopic scales, where the action is on the order of the Planck constant". Now, string theory attempts to unify both classical mechanics and QM, but it has it's own problems, such as having many variants, all of which are untestable. StuRat (talk) 20:06, 8 February 2013 (UTC)[reply]
And now, rather than admitting to being wrong, you go into obfuscation mode. The original question was why don't electrons go into the nucleus. Nothing to do with "scales". You gave the exact anti-answer - a known incorrect answer that was thrown out the window exactly because it was discovered that it leads to electrons going into the nucleus, making all atoms in the universe disappear in a fraction of a second. Your habit of doing this is unfortunate given the volume of text you write on the reference desk. 88.112.41.6 (talk) 20:24, 8 February 2013 (UTC)[reply]
Put up or shut up. I've provided links supporting my point. If you disagree, let's see your proof, not personal attacks. StuRat (talk) 20:29, 8 February 2013 (UTC)[reply]
Holy shit, that's your source? Don't mind if I don't believe a word of what's written on some hand-made website. I've not applied the Crackpot index to it, but my sense tells me it'd be ridiculous. Seriously Stu, if you're going to make a claim like that, you're going to need a better source. The same Wikipedia article on quantum mechanics you cite also states "Predictions of quantum mechanics have been verified experimentally to an extremely high degree of accuracy. According to the correspondence principle between classical and quantum mechanics, all objects obey the laws of quantum mechanics, and classical mechanics is just an approximation for large systems of objects (or a statistical quantum mechanics of a large collection of particles). The laws of classical mechanics thus follow from the laws of quantum mechanics as a statistical average at the limit of large systems or large quantum numbers.[35] However, chaotic systems do not have good quantum numbers, and quantum chaos studies the relationship between classical and quantum descriptions in these systems." (bold mine) I mean really Stu, if you're going to make silly claims like this, don't quote a source (the Wikipedia article) that directly contradicts you. Seriously, read the entire article next time. You might actually learn something new. --Jayron32 20:27, 8 February 2013 (UTC)[reply]
What's that opening line saying about Plank distances, then ? Also, out Theory of everything article says it's "... a theory that would unify or explain through a single model the theories of all fundamental interactions and of all particles of nature: general relativity for gravitation, and the standard model of elementary particle physics — which includes quantum mechanics — for electromagnetism, the two nuclear interactions, and the known elementary particles". So, if you're claiming that QM already does that, then why do we need a ToE ? StuRat (talk) 20:32, 8 February 2013 (UTC)[reply]
Planck distances are the distances when the classical model stops working, which is why we need the quantum model to work there. It has nothing to do with the upper limit for quantum models. The quantum models work on all scales. Also, QM doesn't do everything. QM doesn't have a good explanation for gravity, which is why we need a theory of everything. Gravity is very well modeled by general relativity in the sense that general relativity makes mathematical predictions about gravity that are borne out by experimental data, and which classical explanations of gravity do not. The other fundemental forces are well modeled by quantum mechanics: that is QM makes predictions about how forces like the nuclear forces and electromagnetic forces behave, and those predictions are borne out in the experimental data. The problem is that we have two post-classical theories, each of which very accurately matches experimental data in their own domains (and each does a better job in those domains than classical physics does) which appear to be incompatible with each other. --Jayron32 20:47, 8 February 2013 (UTC)[reply]
It sounds like the lede to our QM article needs changing, then, since it really doesn't say that now. Also, since gravity is critical to understanding macroscopic behavior, if QM doesn't explain it, then QM does have an upper size limit, be that the Plank length or something else. StuRat (talk) 21:22, 8 February 2013 (UTC)[reply]
Furthermore, Sting theory isn't trying to marry classical mechanics and quantum mechanics. It's trying to marry general relativity and quantum mechanics. GR and QM are two different non-classical theories that operate in different domains (GR models gravity, while QM models other forces) --Jayron32 20:31, 8 February 2013 (UTC)[reply]
OK, I was a bit sloppy in equating GR with classical mechanics, but they do rather go together, as GR does work with large objects, like stars and planets, and not-so-much with wave probability functions from QM. I like to think of GR as tweaking classical mechanics, while QM tosses it out the window entirely. StuRat (talk) 20:39, 8 February 2013 (UTC)[reply]
It's a far cry from a mere "tweak" of classical gravity; it also throws classical gravity out the window. Classical gravity has two problems 1) it propagates instantaneously instead of at the speed of light and 2) classical gravity is a real force, whereas in GR gravity is a pseudoforce akin to centrifugal force. That is, General Relativity says that gravity isn't a force at all; it's a geometric warping of space time that gives the observer the 'illusion' of a force. This idea that spacetime can be warped isn't a mere tweak, its the central aspect of GR and is completely and totally out of the realm of classic physics. --Jayron32 20:53, 8 February 2013 (UTC)[reply]
I call it a tweak since it seems mostly to just be another way to visualize gravity. The results are similar, whether you think of space-time as warped or explain orbits with classical mechanics. The speed of light considerations are where more of a difference is observed between the two. StuRat (talk) 21:10, 8 February 2013 (UTC)[reply]
If by "tweak" and "just another way to visualize" you mean "a complete and total rewrite of every aspect of our understanding of how the universe works on the most fundamental levels" then you may be closer to the truth. Seriously Stu, I say this out of love, because I don't want to see you embarrass yourself further along this line of thinking, but you're like a walking, talking personification of the Dunning–Kruger effect. You started with a completely incorrect premise, and rather than concede that you continue to make statements which, bafflingly, are progressively wronger and wronger. GR is not another way to visualize gravity. GR is a fundamentally different way to understand the entirety of physics and motion and kinetics and dynamics and velocity and acceleration and time and space. It has it's own mathematics, it's own geometry, there's absolutely nothing that Newton would look at and say "Yup, that's basically what I said". The results as far as gravity is concerned are not "similar", excepting in the classical limit. At an approximation, the kinds of behaviors that GR predicts about how, say, an apple will move as it descends towards the ground are identical to what classical physics says; this is analogous to the way in which quantum mechanics and classical mechanics converge to the same mathematical results when you get to large objects. However, that doesn't mean that the kinds of differences between the predictions of GR and classical mechanics makes are just minor tweaks. Classical gravity is way off and doesn't correctly model lots of behaviors correctly at all; things like black holes and gravitational time dilation and Frame-dragging and the Geodetic effect which classical gravity gets wrong in the exact same way that classical mechanics gets the atom wrong. There's just no intellectually honest way to say that General Relativity was merely a change of perspective or a tweak to classical gravity: it's a fundamentally different way to look at the universe, and it makes different predictions that match experimental results in ways that classical gravity just does not. --Jayron32 22:16, 8 February 2013 (UTC)[reply]
Look at my example, of planetary orbits. How does GR predict different orbits from classical mechanics ? Newton would definitely say "that's just another way to look at it", not, "I was wrong". It's only when you get far from our everyday experiences that GR predictions are far removed from those of Newton. StuRat (talk) 22:52, 8 February 2013 (UTC)[reply]
Well yes, in general, but even with this GR predicts the correct orbit of Mercury but Newtonian physics does not.--Gilderien Chat|List of good deeds 23:23, 8 February 2013 (UTC)[reply]
Newton gets the orbits of planets wrong by incorrectly predicting the way that they precess. The observed apsidal precession of the planets does not in accurately match the behavior predicted by simple classical mechanics (Newton and Keppler and all that jazz), you need general relativity to predict it correctly. It's wrong for all planets, but noticably so for Mercury; the incorrect predictions about Mercury's orbit are large enough that they were noted before Einstein was even born. No one could explain it until General Relativity created a model that fit perfectly. If you used a GPS in the past week, if it were not for General Relativity, said GPS would have been off by miles in telling you what your position is: based on classical understanding of gravity, your position cannot be accurately calculated using GPS, because classical gravity has no means to calculate Gravitational time dilation which must be taken into account given the distance the GPS sattelite is from the surface of the earth. Newtonian gravity can't explain Gravitational lensing, something astronomers work with every day. Also, no one said Newton was wrong. As I said above, science isn't looking for 'right'. Science is looking for 'useful'. Newton's theories are still, to this day, very useful: most of the stuff you do on a daily basis that may require basic physics calculations, such as the speed your car moves, or the trajectory of a thrown object, or any number of other common calculations, are perfectly well predicted by Newton's theories. Where they fall short, the more complete General Relativity steps in to help expand the corpus of applications, for example the two shortcomings I noted above. Now, General Relativity also predicts the same things about the speed of your car and the flight of a thrown object that classical physics does. It's just silly to do all that math to get the same answer in those sorts of applications. --Jayron32 23:25, 8 February 2013 (UTC)[reply]
I'd still call all that "tweaking". For an example of "a complete and total rewrite of every aspect of our understanding of how the universe works on the most fundamental levels", I'd go with the ancient Greek model of the "elements" being earth, air, fire, water, and ether, each corresponding with a perfect solid, versus our current understanding of the elements. StuRat (talk) 00:13, 9 February 2013 (UTC)[reply]
Apples and oranges. There's no fundamental way you can call the Platonic elements "science". However, the differences that General Relativity wrought on our total understanding of physics is akin to the difference between Dalton's atomic theory and the modern Atomic orbital model of the atom. That is, what Dalton said the atom looked like and what modern Quantum mechanics thinks the atom looks like is roughly akin to the difference between what Newton said the physical universe worked like and what GR says of the same. --Jayron32 00:23, 9 February 2013 (UTC)[reply]
(edit conflict) Damn good question. Since the structure of the atom was first elucidated by Rutherford in his gold foil experiment, that was a major, central controversy with his nuclear model of the atom. They should crash into the nucleus under the principles of classical physics, and there's nothing in classical physics which adequately explains why they don't. Indeed, that problem is one of the keystones which brought classical physics (as a means to describe atomic-level phenomena) down and led to the development of more advanced, modern understandings of the physics. Now, the first thing you need to do is to take the image of the little electron orbiting the nucleus like the earth does the Sun and put it in the same part of your mind that you keep Santa Clause and the Easter Bunny: it's a nice little story we tell kids, but its a total fiction. An electron is not a little ball. Trying to describe an atom as you would describe any other object is always problematic, but if you must create a picture in your mind, it is better to think of an electron as a cloud instead of a point. The cloud exists over a volume of space described by a wave function which describes both the shape and density of that cloud over space. Now, that cloud is imbued with certain properties that must be conserved; that is no change to that cloud can eliminate said properties. The electron cloud has, for example, momentum. Now, this momentum exists spread out over the whole cloud, and cannot be localized to any one point in the cloud (I know, this makes no sense when you try to picture it, but it doesn't need to make sense to be true. This is one of those things you have to be able to accept without a visual representation). Any changes to the cloud, such as its size, must conserve this momentum. If the cloud were entirely compressed into the nucleus, then this momentum would be localized into a single point. The fact that this kind of localization of momentum is impossible is enshrined in the uncertainty principle, which is a cornerstone theorem of modern physics. So the reason that the electron never crashes into the nucleus is that, if it did that, its momentum would be localized in one location, and the uncertainty principle says that you can't localize momentum in that way. The uncertainty principle says any particle cannot simultaneously have a precisely defined location and momentum; since all electrons have a precisely defined momentum, they can NEVER have a precisely defined location. So electrons cannot "crash" into the nucleus.
Another way to think of it is this: there is a force holding the electron to the nucleus in the same way that there is a force holding a magnet to, say, your refrigerator. Now, there is not any loss of energy as the magnet holds fast to the refrigerator. The magnet has potential energy, but so long as the magnet does no work, then it will remain stuck to the refrigerator forever without losing any of that potential energy. Work only happens when a force moves something; forces that don't move objects don't expend any work, and no energy is "used up". In almost the exact same way, the electron is "held fast" to the nucleus by the electrostatic force, but no energy is lost because the electron does no work in remaining there. Here's where the classical explanation and modern explanation diverge: if you're picturing the electron as a little ball, there's no way for it to remain in motion under such a force and do no work at all. Such a ball should spiral into the nucleus, shedding potential energy the whole way in. If, however, you think of the electron as a steady-state cloud it just remains in the same state forever, and does no work. Now, the mindfuck here is that you need to accept that this steady state cloud still has momentum without actually having any localized parts which are in motion. There's no convenient way to make that work except to say "just accept it". --Jayron32 19:00, 8 February 2013 (UTC)[reply]
Well technically in hydrogen it sort of does - the 1s electron is in an orbital which is a sphere shape and superimposed over the proton, which is also technically a sphere of probability. So under some observations, it will appear to be "inside" the nucleus.--Gilderien Chat|List of good deeds 20:20, 8 February 2013 (UTC)[reply]
And keep in mind that the electron has half a spin all by itself, in addition to the full spin of orbiting around the nucleus. So the final result is either a spin and a half if aligned, or half a spin if opposed and no other result can ever be measured. Hcobb (talk) 20:23, 8 February 2013 (UTC)[reply]
As is multiply explained above and in the linked articles, "spin of orbiting around the nucleus" is a load of completely disproven nonsense, no? DMacks (talk) 22:06, 8 February 2013 (UTC)[reply]
Well as I read it, the "cloud" does have momentum and this would thus be either in one direction or another (as a non-zero vector quantity must be) and so these effects will be observed.--Gilderien Chat|List of good deeds 23:35, 8 February 2013 (UTC)[reply]

(arbitrary break) Why is "Why don't electrons fall into the nucleus?" the single most asked science ref desk question ever?[edit]

self explanatory... μηδείς (talk) 20:43, 8 February 2013 (UTC)[reply]

Well, two things 1) It isn't (the search you gave gives LOTS of threads that mention the words "electron" and "nucleus" without addressing this specific problem) and 2) If it is a commonly asked question, that's because of what I explained above: things like atoms, electrons, light, nuclei, etc. etc. don't have analogues to what you can sense and experience in the world. There is literally no shape, object, behavior, or concept about how you experience the world with your five senses which correctly and accurately models quantum behavior. That is, the predictions you would make about an electron if you say "an electron is like FOO", where FOO is literally anything you have ever experienced, always breaks down. Some representations work in some situations, which is why we use them in some explanations, but no such representation works all that well. So, you need to rely solely on two things A) the data from experiments and measurements and B) the predictions of the equations of quantum mechanics. As long as A = B, it's a highly useful theory. The fact that it's also not a theory that lends itself to easy visualization doesn't invalidate it; but it does make it very hard for people to wrap their heads around, since we all learn by analogy; we try to connect something we're learning to something we already know. QM resists that sort of analogy, which is why it is so confusing, and why people have a hard time wrapping their heads around it. --Jayron32 21:11, 8 February 2013 (UTC)[reply]
I wasn't challenging any prior answers, just pointing out the same question was asked about a month ago and several other times I remember. A search on "fall into the nucleus" gets you lots of prior answers, only a few of which are links to Dr Who episodes. μηδείς (talk) 02:53, 9 February 2013 (UTC)[reply]
I wanted to comment on a few things, but didn't want to insert my comments into the above mess. So I'm putting all of them here.
First, why the heck are we debating whether quantum mechanics works for classical systems? It does, period, end of story. In fact, we have an article about this: correspondence principle, which gives plenty of examples. In any undergrad QM class, you get to prove ad nauseum how your quantum result for various problems reduces to the classical result in the limit of high energies and large sizes. If it doesn't, then you did something wrong, and have to redo the problem. QM is incompatible with general relativity because the former cannot describe strong gravitational fields, not because it can't describe large sizes. It's quite easy to describe weak gravitational fields in QM: just introduce a classical 1/r gravitational potential. Strong gravitational fields usually arise at very small scales, like the singularity of a black hole or the very early universe, not at large scales like that of the solar system.
Second, in the classical model where the electron orbits a positive nucleus, the electron would radiate energy and crash into the nucleus. As Jayron noted, this fact is expressed in the Larmor formula. This, again, is indisputable. Bohr model#Origins describes the historical significance of this (although strangely, Rutherford model doesn't):
"Rutherford naturally considered a planetary-model atom, the Rutherford model of 1911 – electrons orbiting a solar nucleus – however, said planetary-model atom has a technical difficulty. The laws of classical mechanics (i.e. the Larmor formula), predict that the electron will release electromagnetic radiation while orbiting a nucleus. Because the electron would lose energy, it would gradually spiral inwards, collapsing into the nucleus. This atom model is disastrous, because it predicts that all atoms are unstable."
Third, here's an alternative answer to the OP's question, which is equivalent to Jayron's in the sense that both use the same underlying theory. In quantum mechanics, the electron is represented as a wavefunction. The physical significance of the wavefunction is that if you try to measure the electron's position, the square of the wavefunction at any point is equal to the probability of finding the electron at that point. Schrodinger's equation tells you what the wavefunction can possibly look like. So, why is it not possible for the wavefunction to be extremely densely concentrated around the nucleus? Because such a wavefunction doesn't satisfy Schrodinger's equation. You can see every function that does satisfy the equation; they're called atomic orbitals. Note, however, that the squared wavefunction is only a probability distribution, and even very close to the nucleus, it doesn't fall to 0. You might ask, if this is the case, why electrons don't sometimes fall into the nucleus. The answer is that they do, and this process is called electron capture.
Finally, contrary to what Jayron said, electron clouds have no momentum. This is because atomic orbitals are stationary states, and the expected value of the momentum operator is always 0 in any stationary state. The expected value of the momentum squared, however, is non-zero, so electrons have non-zero kinetic energy. --140.180.247.198 (talk) 00:35, 9 February 2013 (UTC)[reply]
Thanks for correcting me on the momentum/kinetic energy thing. I confused the terms above in my initial explanation. However I am still pretty confident in the uncertainty principle implications of having an electron located in the nucleus. I did some more digging just now to check up on the other aspects of my explanation, and found this explanation from the University of Illinois physics department, which may be helpful towards answering the initial question. --Jayron32 00:47, 9 February 2013 (UTC)[reply]
Well, I am not a expert of Quantum Mechanics and the above discussion is completely based on Quantum Mechanics. I have thought a answer to my question but I think that is wrong. "Suppose, an electron gets attracted to the nucleus, it falls towards the nucleus, coming to a lower orbit and then to other lower orbits between the electrons's own orbit and the nucleus. We know, for an electron to come to a lower orbit, it would have to emit photon. The attraction of nucleus for an electron is not so strong that it will make an electron to emit photon. Therefore, the electron doesn't come to a lower orbit. This is why electrons don't fall in the nucleus". Am I right or wrong? Want to be Einstein (talk) 03:46, 9 February 2013 (UTC)[reply]
See my answer in this old thread. GilderienJayron32 made some similar points earlier in this thread. -- BenRG (talk) 06:02, 9 February 2013 (UTC)[reply]
Here is some questions related to your question- First, second, third, fourth and Geiger–Marsden experiment. Polar Bear25 (talk) 11:51, 10 February 2013 (UTC)[reply]

How to freeze water in 5 seconds?[edit]

Watch this video I know there is a way to freeze water in 5 seconds. But I do not know how to do it. After watching the linked video please tell me a way by which I can also make water freeze very quickly. Polar Bear25 (talk) 19:57, 8 February 2013 (UTC)[reply]

Well, there are four factors on how to freeze water quickly, in general (although this seems to be a case of supercooling, see answer below):
1) Have it as close to freezing temperature as possible, before you start.
2) Expose it to extreme cold.
3) Make the water particles as small as possible.
4) Lower the pressure.
So, if you spray a fine mist of high-pressure, near-freezing water through liquid nitrogen, while lowering the temperature, you should be able to freeze it considerably quicker than 5 seconds. StuRat (talk) 20:09, 8 February 2013 (UTC)[reply]
(ec) It's called supercooling. It's a trick in which purified water is cooled down very carefully below freezing point so that it gets no 'opportunity' to crystallize. If the water is disturbed, it then suddenly starts to form ice crystals. When you see the water freezing, its temperature does not actually decrease, but it increases because energy is released by the formation of strong bonds in the crystal. - Lindert (talk) 20:14, 8 February 2013 (UTC)[reply]
How can I do this as the person in the video did ? Polar Bear25 (talk) 03:57, 9 February 2013 (UTC)[reply]
There are plenty of videos on the web, e.g. this one. For best results use distilled or spring water, and leave in freezer for about three hours.--Shantavira|feed me 10:18, 9 February 2013 (UTC)[reply]
Note that spring water is not suitable because it is very far from pure. See Spring_water#Water_content. 202.158.103.42 (talk) 14:47, 9 February 2013 (UTC)[reply]
You are on the right path. Placing bottle in freezer water, but how will I freeze it. Polar Bear25 (talk) 11:09, 9 February 2013 (UTC)[reply]
Wash and rinse the bottles carefully, rinsing the last couple times with distilled water. You need to have the timing right, so they are below freezing, but not too much. Or the freezer temperature needs to be set just below freezing temp. Then, just shaking one should make it freeze. An alternative is to remove the lid. The lowered pressure may also trigger freezing, so it could freeze before you get the lid off.
However, note that there's considerable luck involved. Some stray particle which acts as a nucleation site for ice often gets into the water. So, many bottles of water would give you a better chance that some become supercooled, and, when some start to form ice, that's a good temperature indication, so, if you see others interspersed with the frozen ones which aren't frozen, then you could shake them to try to get a quick freeze out of them
One word of caution: Be sure to fill the bottles only maybe 2/3 full. The air gap is needed since water expands when it freezes, and needs someplace to expand into, or a glass bottle might shatter. StuRat (talk) 16:58, 11 February 2013 (UTC)[reply]

Space between atoms[edit]

In the article atomic spacing it says that the distance between atoms varies from a few angstroms in solids to "as large as a meter" in "outer space". Yet I asked a similar question before here and was told that the distance between the inner electron and the nucleus could technically be infinite. The wording in the article also implies that if the distance between 2 or more atoms can be measured then the distance between the electron cloud and the nucleus can also be measured. Is there a rule for determining the distance between the inner electron cloud and the nucleus of an atom?165.212.189.187 (talk) 20:38, 8 February 2013 (UTC)[reply]

"The cloud" is just a large space where the electron "probably or usually" is. All we can actually say is how likely it is for the electron to be a certain distance away, or else to say "the electron is unlikely to be [somewhere]" for some essentially arbitrary meaning of "unlikely" (there's not literally an "inner edge" of the cloud the way your question suggests). DMacks (talk) 21:13, 8 February 2013 (UTC)[reply]
There is no such thing as an "infinite" distance. But they might mean that there is no limit to how far the distance could be, which is not quite the same thing mathematically. ←Baseball Bugs What's up, Doc? carrots→ 22:10, 8 February 2013 (UTC)[reply]
To put it in context, it's overwhelmingly likely that the electrons are closer to the nucleus than the nucleus of the next atom along - but there is a vanishingly small chance that the electron could be over on the other side of the universe. There is no particular paradox to this. The "distance between atoms" is likely to be measured from nucleus to nucleus. SteveBaker (talk) 00:08, 9 February 2013 (UTC)[reply]

Is "the" electron in a hydrogen atom really any random electron in existence and not necessarily the same over time?GeeBIGS (talk) 02:48, 9 February 2013 (UTC)[reply]

Why isn't empty space listed as a critical component of an atom? Since without it you can't have an atomGeeBIGS (talk) 02:54, 9 February 2013 (UTC)[reply]

Space would be a "component" of everything that exists, so it's kind of "understood". ←Baseball Bugs What's up, Doc? carrots→ 06:17, 9 February 2013 (UTC)[reply]

It's not a component of an electron. Quarks could also be a component of everything that exists,right? but you don't just take them for granted or as you call it understood, what's the difference?GeeBIGS (talk) 07:08, 9 February 2013 (UTC)[reply]

IIRC, atoms and the like are pretty much localized to planets, etc.; most of the matter in "outer space" is more accurately Plasma_(physics)#Common_plasmas. (i.e., nuclei and electrons are just floating around promiscuously instead of in committed relationships) Gzuckier (talk) 04:39, 11 February 2013 (UTC)[reply]

DVI and VGA cable[edit]

Using a DVI cable instead of a VGA cable makes any difference in quality, performance or anything?
…using the same monitor(this monitor)
thanksIskánder Vigoa Pérez (talk) 22:52, 8 February 2013 (UTC)[reply]
The Computer Ref Desk might be a better place to post this Q. StuRat (talk) 23:02, 8 February 2013 (UTC)[reply]
VGA is analog, DVI is digital. If you use VGA the monitor will convert it to digital internally, so it can't look any better than DVI. Whether it will look worse depends on the quality of the analog-to-digital converter (and how picky you are). I've seen wide variation in this, and I can't tell you anything about that particular monitor. As far as performance goes, it won't have any effect on frame rate. There could be a tiny difference in the lag time between the video card emitting the signal and the monitor displaying it, but I don't know which interface it would favor. -- BenRG (talk) 05:45, 9 February 2013 (UTC)[reply]
Use DVI if you can. If the source can produce high definition video it may only do that for DVI or the image may be slightly degraded comparatively even if it does send it over a VGA cable. Dmcq (talk) 11:56, 9 February 2013 (UTC)[reply]

Amount of heat input required to cut stainless steel plates[edit]

The first row of the table in this section seems highly counter-intuitive. If I'm reading it correctly, it seems to state that as plate thickness increases you actually need less laser power to cut through it. I've check the referenced source[12] and it is not a typo. Is this actually true? I'm having a hard time believing it despite the authoritative source. Is there an explanation for this? Dncsky (talk) 23:47, 8 February 2013 (UTC)[reply]


That does seem a little odd at first sight. I actually own a laser cutter (a 120Watt lasersaur)...and although I don't use it to cut metal - I think I may have some insight here.
I know that when cutting acrylic plastic with my laser cutter, the slot that the laser cuts acts like a waveguide - keeping the beam tightly in focus rather than spreading out. That means that you don't need much (if any) additional laser power to cut thick acrylic than thin...within reason.
When cutting wood and metal, it's common to blast a "cutting gas" into the slot where the laser is cutting - for cutting wood, you can use nitrogen to exclude oxygen from the cut and thereby prevent scorching. Or you if you're a cheapskate like me you can just blast air into the slot instead! A lot of people who are cutting metal use pure oxygen as their cutting gas. The gasses inside the cut become insanely hot and this "gas assist" lasering process pushes that hot gas into the path of the laser so that the material is pre-heated by the time the laser reaches it.
When you're cutting a thick, non-combustible material, the gas assist works much more effectively than for thin materials - and for shiney materials, the "waveguide" effect keeps the laser well focussed throughout the thickness - which is another big win. With metals, a consequence of that is that you need a heck of a lot of laser power to make that first hole (the "pierce") - but after that, you can dial it way back for the actual cutting. I'd be surprised if you could cut steel at all with a 250 watt laser without pre-heating it enough to get that first pierce hole and allow the waveguide and hot gas channelling to start working for you.
But here is the key: The business of delivering laser power onto the target is as much a matter of speed as power. My laser (which I predominantly use for cutting plywood) can cut effectively at a speed of 2,000 mm/sec only if the laser is operating at 100% full power. But you can also cut the same thickness of plywood at 50% power if you drop the feed rate to around 800 mm/sec. In effect, you're holding the laser in one place for longer so that the total energy delivered to the material is about the same. Balancing speed versus power is important for some materials that are liable to burn or melt or buckle when they get too hot.
So I suspect that although the laser power can be much smaller for thicker materials, the feed rate is probably a heck of a lot slower too.
In fact, that's backed up beautifully by our article - which shows 1,000 inches per minute as the feed rate for the thinnest steel and just 18 inches per minute for the thick stuff! In terms of Watt-seconds per linear inch of material cut (now that's a weird unit!), then using the data from the table, for the very thin steel, it's taking 1,000 Watts for 60 seconds (60,000Ws) to cut 1,000 inches of thin steel. That's 60Ws per inch of cutting. For the thickest steel, you need 250 Watts for a 60 seconds (15,000Ws) to cut for just 18" - which is 833Ws per inch. In other words, you need about 13 times as much laser energy (measured in Watt-seconds) to cut a 1" long slot in 0.25" steel than you do to cut the same slot in 0.02" steel...which (un-amazingly) is the same amount of energy per square inch of material removed - no matter the thickness.
The table is really saying that for very thick material, you can cut MUCH more slowly - and if you do that, you can get away with less laser energy. Doubtless you could cut 0.02" steel with a 250 Watt laser if you went slowly enough...and 0.25" steel would cut more quickly with a 1,000 Watt laser. But going so slowly with a thin sheet of material might produce other problems such as buckling and undesired melting away from the edge of the cut-line. So probably you have to use a lot of power to cut it quickly and thereby avoid those kinds of problem. Thicker material is unlikely to buckle - so you can take your time and use a smaller laser.
SteveBaker (talk) 00:42, 9 February 2013 (UTC)[reply]
Perfect answer. Thanks a lot!Dncsky (talk) 01:27, 9 February 2013 (UTC)[reply]
It looks like your cutting gas has a couple of uses. The Wood/nitrogen case is clearly to stop combustion, but the steel/oxygen case turns the laser cutter into a Oxyfuel cutting torch. One of those can cut steel up to 2 feet (0.66 meters) thick because the steel itself works as a fuel. --Guy Macon (talk) 03:53, 9 February 2013 (UTC)[reply]
Yes, I believe so. When I was considering using Nitrogen as a cutting gas on my lowly 120 Watt beast, I wanted to work out how long a $100 cylinder of the stuff would last me (answer: Not long enough!) - and I got into an online discussion with a guy who was actually talking about cutting thick steel. He came up with numbers that implied to me that he was using a gas stream at close to the speed of sound - blasting down through the same hole that the laser beam emerges from - so it's a properly coaxial affair. Clearly that amount of pressure with pure oxygen and with all the heat that several kilowatts of laser energy adds will indeed produce something very similar to an oxyfuel system.
I soon realised that he and I were talking about entirely different systems! My machine produces a perfectly useful "air-assist" for cutting 3mm plywood using nothing more than a $77, 0.7 cubic feet per minute airbrush compressor - feeding a coaxial air stream through a 5mm hole...not much more than a gentle breeze! But even that relatively modest gas jet produces a dramatically better cutting speed than the laser alone (and incidentally blows smoke and debris away from the horribly expensive gold coated zinc-selenium lens). So there is quite a bit of subtlety to how these systems work.
Even on my lasersaur, the means by which the laser cuts varies from material to material. When cutting wood with an air jet, it combusts the material, producing smoke, water vapor, carbon dioxide, some kind of yellowish tar-like substance that coats my machine and has to be cleaned off regularly. It leaves behind a very smooth, but somewhat charred, edge. However, if you do use nitrogen to prevent combustion, the wood just "goes away" without producing smoke and the edge of the material is very clean. When cutting acrylic plastic, the laser produces some kind of chemical change that's not just combustion - and the result is a perfectly clean cut with shiny, smooth, clear edges. With some other plastics, it simply melts the material without any chemical change whatever. SteveBaker (talk) 16:32, 9 February 2013 (UTC)[reply]
Wait: A feed rate of "2,000 mm/s"? Wow! What a nice laser you have there. Caristan, C L (2004) Laser Cutting Guide for Manufacturing. SME. p. 23 suggests a 2 kW CO2 (pulse mode) laser cuts 1 mm stainless steel sheet at closer to 133 mm/s (8 m/min) --Senra (talk) 20:54, 9 February 2013 (UTC)[reply]
Ack! Sorry - that should have been 2000mm per MINUTE not per second! But in any case, I'm not cutting metal - thin plywood, acrylic plastic, cloth, paper, cardboard, etc. You can see the stuff we make with it at http://renaissanceminiatures.com SteveBaker (talk) 04:58, 10 February 2013 (UTC)[reply]